LSAT and Law School Admissions Forum

Get expert LSAT preparation and law school admissions advice from PowerScore Test Preparation.

General questions relating to LSAT Logical Reasoning.
 legalrabbit
  • Posts: 8
  • Joined: Sep 01, 2011
|
#1756
I'm trying to internalize the differences when approaching assumption questions.... I think I have a decent handle on sufficient questions, but I'm sort of lost on necessary. Could someone please help me out?


Basic rule: sufficient condition --> necessary condition.

If the Q stem is asking for the sufficient condition, and it gives us
Premise: A --> B
Conclusion A --> C

What are the possible correct answer types?
Sufficient: B --> C or its contrapositive, not C --> not B

Trap incorrect answers:
A --> B (doesn't talk about C)
its contrapositive not B --> not A
C --> B (backwards)


If the Q stem is asking for the necessary condition and it gives us
Premise: A --> B
Conclusion: A --> C

What are the possible correct answer types?

What are the possible trap answers?
User avatar
 Dave Killoran
PowerScore Staff
  • PowerScore Staff
  • Posts: 5853
  • Joined: Mar 25, 2011
|
#1767
If the Q stem is asking for the necessary condition and it gives us
Premise: A --> B
Conclusion: A --> C

What are the possible correct answer types?

What are the possible trap answers?
The possible correct answers (Assumptions) are:

B --> C or the contrapositive of that statement.


There are numerous trap answers, including:

C --> B or the contrapositive (these two are the most likely wrong answers)
B --> A or the contrapositive
C --> A or the contrapositive


Remember, in many instances, what is an assumption of the argument and what justifies the argument overlap.

Does that make sense?
 legalrabbit
  • Posts: 8
  • Joined: Sep 01, 2011
|
#1779
Thanks for the explanation. I was hoping that there was a key difference but it looks like diagramming doesn't show big differences when it comes to sufficient and necessary assumption questions.


This is tangential to my original question, but in the Powerscore's Weaken chapter, it states that when the stimulus has a conditional conclusion, one should immediately "attack the necessary assumption."

Could you please rephrase that? What exactly should I be doing?
User avatar
 Dave Killoran
PowerScore Staff
  • PowerScore Staff
  • Posts: 5853
  • Joined: Mar 25, 2011
|
#1781
The diagramming doesn't show a big difference in this instance, but that doesn't mean there aren't further differences here. The key difference is that you could add just about anything to the Justify answers and you would still have a correct answer, but if you add anything to the Assumption answers they will be wrong.

For example, if the correct answer to two different Justify and Assumption questions was B --> C in both cases, you could add D ( or E, or F, etc) to the Justify answer and it would still work, as in:

B --> C + D

That answer would still be correct for a Justify answer, but in the assumption question the presence of D would cause the answer to be incorrect. That is one element that makes these questions different, and makes Justify answers, at times, more challenging to identify.

In the LRB Weaken chapter, my meaning is that you attack the idea that some element is necessary to a part of the argument. For example, if we have A --> B in the argument's conclusion, attack the idea that B is indeed necessary. That is one method of attacking these questions (see the Carpet Market question in that chapter for a prime example).

Does that help?

Get the most out of your LSAT Prep Plus subscription.

Analyze and track your performance with our Testing and Analytics Package.